What is the cost, in dollars, of heating the hot tub, assuming 75.0% efficiency to account for heat transfer to the surroundings?

Answers

Answer 1

Answer:

$ 5.93

Explanation:

If we are to put into consideration the following factors even though it was not all stated in the question.

What is the cost of heating a hot tub containing 1500 kg of water from 10.0°C to 40.0°C, assuming 75.0% efficiency to take heat loss to surroundings into account? The cost of electricity is ¢8.50/kW·h.

Q = mc∆T .........1

Where m is the mass of water = 1500kg

C is the specific heat capacity of water = 4.184

∆T = temperature change

= 40° - 10°

= 30°

Now, substitute the values into equation 1

= 1500×4.184×30

= 188280

Q in kw/s

= 188280/3600

= 52.3kw/hr

Efficiency = 75%

= 75/100

= 0.75

Hence we have

52.3/0.75

= 69.73kw/hr

C = Q × cost per kw/hr

C = 69.73× $8.5/100

C = 5.92705

C =$ 5.93

Hence, the cost of heating the hot tub is $ 5.93


Related Questions

Jeni walks 100 meters east and then 50 meters north. How big is Jeni's displacement from the starting point?
a. 100 meters
b. 150 meters
c. 50 meters
d. About 112 meters

Answers

Answer:

d. About 112 meters.

Explanation:

From the question, John's displacement forms a right angle triangle as below.

Using Pythagoras theorem,

a² = b²+c²....................... Equation 1

Where a = John's displacement from the starting point, b = 100 m, c = 50 m

Substitite these values into equation 1

a² = 100²+50²

a² = 10000+2500

a² = 12500

a = √12500

a = 111.8 meters.

a = about 112 meters.

The right answer is d. About 112 meters.

Show two data points from your simulation that demonstrate this behavior.

I1 V1 I2= 2I1 V2=2V1 V1/ I1 =V2/I2

For the light bulb, why is it better to take more measurements in the range 20mA < I < 40mA, instead of just taking equally spaced measurements in the entire range of 0 mA < I< 55mA

Answers

Answer:

hello your question is incomplete attached below is the complete and the required circuit diagrams

answer :

Ai) This proves that when the current across the resistor is doubled the value of the voltage across the resistor doubles as well

B) It is better to take more measurements in the range 20mA < I < 40mA because of the amount of temperature reached by the bulb and the change in resistance is affected by the temperature

hence At 0 mA current, there won't be any noticeable change

Explanation:

Ai) The voltage across the resistor will double when you double the current through the resistor

Given that : V = I*R.  

lets assume : I = 2 amperes , R = 3 ohms

V = 2*3 = 6 v

secondly lets assume double the value of  (I)   i.e. I = 4 amperes

hence : V = 4*3 = 12 volts

This proves that when the current across the resistor is doubled the value of the voltage across the resistor doubles as well

Aii) Showing the two data points from simulation

I1                    V1            I2= 2I1         V2=2V1       V1/ I1 =V2/I2

0.9*10^3     9 * 10^3     1.8*10^3       18*10^3          10 ohms

1.6 * 10^3    16 * 10^3    3.2*10^3     32*10^3         10 ohms

B) It is better to take more measurements in the range 20mA < I < 40mA because of the amount of temperature reached by the bulb and the change in resistance is affected by the temperature

hence At 0 mA current, there won't be any noticeable change

Please help! THIS IS A EASY ONE, HOPEFULLY.

Answers

Answer:

megaliter > kiloliter > liter >centiliter >mililiter > deciliter > nanoliter

Explanation:

plz mark brainlest

.................,,,,,,,,,,,

Answers

Answer:

B

Explanation:

Motion is movement, the teacher's movement is motion

How would you feel if everyone hated you?

Answers

Really sad try not to let it get to you

Answer:

awful

Explanation:

can you help me with my question Which of the themes of Hawthorne's "Dr. Heidegger's Experiment" is illustrated by this passage? Paragraph 36: The most singular effect of their gayety was an impulse to mock the infirmity and decrepitude of which they had so lately been the victims. A. Gayety produces an impulse to mock. B. It is a great release to look back on our problems happily C It is human nature to mock or make light of problems we have been delivered from

Draw a conclusion, based on the solubility curves shown above, of which compound would have the greatest
percentage recovered after cooling a saturated solution of that compound from 90°C to 30°C?
A) KCL
B) NaNO3
C) Nacl
D) KNO3

Answers

Answer: The answer is D. KNO3

Explanation:

The graph shows that the KN03 going straight up from the temperature sign so you reversed that so that it will make it to 90°C to 30°C

To solve this we must be knowing each and every concept related to solubility. Therefore, the correct option is option D among all the given options.

What is solubility?

The greatest amount of one material that may be dissolved in the other is referred to as its solubility. It is the most solute that may be dissolved into a solvent near equilibrium, resulting in a saturated solution.

When specific circumstances are satisfied, more solute can be dissolved further than the solubility limit point, resulting in a supersaturated solution. Adding extra solute after saturation or supersaturation does not enhance the concentration in the solution. Rather, the excess solute begins to precipitated out of solution. KNO[tex]_3[/tex] is the compound that would have the greatest percentage recovered after cooling a saturated solution of that compound from 90°C to 30°C.

Therefore, the correct option is option D.

To know more about solubility, here:

https://brainly.com/question/14366471

#SPJ2

What force is needed to give a 4800.0 kg truck an acceleration of 6.2 m/s2 over a level road? ​

Answers

Answer:

the force needed to give the truck the acceleration is 29,760 N.

Explanation:

Given;

mass of truck, m = 4800 kg

acceleration of the truck, a = 6.2 m/s²

The force needed to give the truck the acceleration is calculated as;

F = ma

F = 4800 x 6.2

F = 29,760 N

Therefore, the force needed to give the truck the acceleration is 29,760 N.

A truck driver is attempting to deliver some furniture. First , he travels 8 km east, and then he turns around and travels 3 km west. Finally, he turns again and travels 12 km east to his destination. a- what distance has the driver traveled? b- what is the drivers total displacement?

Answers

Answer : 17 km this is the total displacement

Balanced forces acting on an object keeps it at ____or moving at _____in straight line.

Fill in the ___ spaces

Answers

Answer:

Please see below as the answer is self explanatory.

Explanation:

According to Newton's 2nd law,  a net force acting on an object of mass m, causes the object to be accelerated.If the forces acting on the object are balanced, which means that the net force on the object is zero, just applying the same law, we find that the object is not accelerated.According to Newton's First law, an object that is not accelerated is at rest, or moves along a straight line at constant speed.So, if there are balanced forces acting on the object, if the object is at rest, will keep at rest, and if it is moving, it will keep moving at constant speed along a straight line.

The Great Sandini is a 60 kg circus performer who is shot from a cannon (actually a spring gun). You don't find many men of his caliber, so you help him design a new gun. This new gun has a very large spring with a very small mass and a force constant of 1300 N/m that he will compress with a force of 6500 N. The inside of the gun barrel is coated with Teflon, so the average friction force will be only 50 N during the 5.0 mm he moves in the barrel.

Required:
At what speed will he emerge from the end of the barrel, 2.5 mabove his initial rest position?

Answers

Answer:

22m/s

Explanation:

Mass, m=60 kg

Force constant, k=1300N/m

Restoring force, Fx=6500 N

Average friction force, f=50 N

Length of barrel, l=5m

y=2.5 m

Initial velocity, u=0

[tex]F_x=kx[/tex]

Substitute the values

[tex]6500=1300x[/tex]

[tex]x=\frac{6500}{1300}=5[/tex]m

Work done due to friction force

[tex]W_f=fscos\theta[/tex]

We have [tex]\theta=180^{\circ}[/tex]

Substitute the values

[tex]W_f=50\times 5cos180^{\circ}[/tex]

[tex]W_f=-250J[/tex]

Initial kinetic energy, Ki=0

Initial gravitational energy, [tex]U_{grav,1}=0[/tex]\

Initial elastic potential energy

[tex]U_{el,1}=\frac{1}{2}kx^2=\frac{1}{2}(1300)(5^2)[/tex]

[tex]U_{el,1}=16250J[/tex]

Final elastic energy,[tex]U_{el,2}=0[/tex]

Final kinetic energy, [tex]K_f=\frac{1}{2}(60)v^2=30v^2[/tex]

Final gravitational energy, [tex]U_{grav,2}=mgh=60\times 9.8\times 2.5[/tex]

Final gravitational energy, [tex]U_{grav,2}=1470J[/tex]

Using work-energy theorem

[tex]K_i+U_{grav,1}+U_{el,1}+W_f=K_f+U_{grav,2}+U_{el,2}[/tex]

Substitute the values

[tex]0+0+16250-250=30v^2+1470+0[/tex]

[tex]16000-1470=30v^2[/tex]

[tex]14530=30v^2[/tex]

[tex]v^2=\frac{14530}{30}[/tex]

[tex]v=\sqrt{\frac{14530}{30}}[/tex]

[tex]v=22m/s[/tex]

what is diffrence between damping and undamping?​

Answers

Answer:

Oscillation whose amplitude reduce with time are called damped oscillation. This happen because of the friction. In oscillation if its amplitude doesn't change with time then they are called Undamped oscillation

Damped and undamped vibration refer to two different types of vibrations. The main difference between damped and undamped vibration is that undamped vibration refer to vibrations where energy of the vibrating object does not get dissipated to surroundings over time, whereas damped vibration refers to vibrations where the vibrating object loses its energy to the surroundings.

The spring is unstretched at the position x = 0. under the action of a force p, the cart moves from the initial position x1 = -8 in. to the final position x2 = 5 in. determine (a) the work done on the cart by the spring and (b) the work done on the cart by its weight.

Answers

This question is incomplete, the missing diagram is uploaded along this Answer below.

Answer:

a) the work done on the cart by the spring is 4.875 lb-ft

b) the work done on the cart by its weight is - 3.935 lb-ft

Explanation:

Given the data in the question;

(a) determine the work done on the cart by the spring

we calculate the work done on the cart by the spring as follows;

[tex]W_{spring}[/tex] = 1/2×k( [tex]x^{2} _{1}[/tex] - [tex]x^{2} _{2}[/tex] )

where k is spring constant ( 3 lb/in )

we substitute  

[tex]W_{spring}[/tex] = 1/2 × 3( (-8)² - (5)² )      

[tex]W_{spring}[/tex] = 1/2 × 3( 64 - 25 )

[tex]W_{spring}[/tex] = 1/2 × 3( 39 )

[tex]W_{spring}[/tex] = 58.5 lb-in

we convert to pound force-foot

[tex]W_{spring}[/tex] = 58.5 × 0.0833333 lb-ft

[tex]W_{spring}[/tex] = 4.875 lb-ft

Therefore, the work done on the cart by the spring is 4.875 lb-ft

b) the work done on the cart by its weight

work done by its weight;

[tex]W_{gravity}[/tex] = -mgsin∅( x₂ - x₁ )        

we substitute in of values from the image below;

[tex]W_{gravity}[/tex] = -14 × sin(15°)( 5 - (-8) )  

[tex]W_{gravity}[/tex] = -14 × 0.2588 × 13

[tex]W_{gravity}[/tex] = -47.1  lb-in

we convert to pound force-foot

[tex]W_{gravity}[/tex] = -47.1 × 0.0833333 lb-ft

[tex]W_{gravity}[/tex] = - 3.935 lb-ft

Therefore, the work done on the cart by its weight is - 3.935 lb-ft

a) the work done on the cart by the spring is 4.875 lb-ft.

b) the work done on the cart by its weight is - 3.935 lb-ft.

Calculation of the work done:

a. The work done on the cart by the spring is

= 1/2 × 3( (-8)² - (5)² )      

= 1/2 × 3( 64 - 25 )

= 1/2 × 3( 39 )

= 58.5 lb-in

Now we have to convert to pound force-foot

So,

= 58.5 × 0.0833333 lb-ft

= 4.875 lb-ft

b) Now

work done by its weight;

= -mgsin∅( x₂ - x₁ )        

So,

= -14 × sin(15°)( 5 - (-8) )  

= -14 × 0.2588 × 13

= -47.1  lb-in

Now we convert to pound force-foot

= -47.1 × 0.0833333 lb-ft

= - 3.935 lb-ft

Learn more about spring here: https://brainly.com/question/16060099

You and a friend each hold a lump of wet clay. Each lump has a mass of 25 grams. You each toss your lump of clay into the air, where the lumps collide and stick together. Just before the impact, the velocity of one lump was < 4, 4, -3 > m/s, and the velocity of the other lump was < -2, 0, -7 > m/s.
What was the the total momentum of the lumps just before the impact?
p(total) = ____kg·m/s.
What is the momentum of the stuck-together lump just after the collision?
p = ____kg·m/s.
What is the velocity of the stuck-together lump just after the collision?
v_f = ____m/s.

Answers

Answer:

a) p(total) = <0.05, 0.1, 0.1 > kg m/s

b) p = <0.05, 0.1, 0.1 > kg.m/s

c) v_f = < 1, 2, 2 > m/s

Explanation:

a.)

Mass of each lump = 25 g = 0.025 kg

Velocity of lump 1 = < -2, 0, -7 > m/s

Momentum of lump 1 = Mass×Velocity

                                   = 0.025×< -2, 0, -7 >

                                   = < -0.05, 0, 0.175> kg m/s

Velocity of lump 2 = < 4, 4, -3 > m/s

Momentum of lump 2 = Mass×Velocity

                                    = 0.025×< 4, 4, -3 >

                                    = < 0.1, 0.1, -0.075> kg m/s

Total momentum before impact  =  < -0.05,  0,  0.175 > + < 0.1, 0.1, -0.075>

                                                      = < 0.05, 0.1, 0.1 > kg m/s

⇒p(total) = <0.05, 0.1, 0.1 > kg m/s

b)

As we know that,

By the law of conservation of linear momentum,

The total momentum will be the same before and after the collision.

⇒Momentum of the stuck together  after the collision = Total momentum of the lumps just before impact.

⇒ p = <0.05, 0.1, 0.1 > kg m/s

c)

Let the final velocity =  v_f

Total mass = 0.025 + 0.025 = 0.05 kg

As

Momentum = mass ×velocity

⇒ <0.05, 0.1, 0.1 > = 0.05 ×v_f

⇒ v_f = <0.05, 0.1, 0.1 > / 0.05

          = < 1, 2, 2 > m/s

⇒v_f = < 1, 2, 2 > m/s

The variable ______________ describes how quickly something moves.

Answers

it's up in Gogle trust me

Assuming the speed of sound is 340 m/s, what is the most likely speed of the jet shown below?

Answers

Well we know it has to be greater than 300,000 km/s since we can't see it.

We can't calculate it any closer than that using the given information.

How much force will a 5 kg rock hit the Earth with if it falls
for 1 second?

Answers

Answer:

f

Explanation:

f

Two charges, one +Q and the other −Q, are held a distance d apart. Consider only points on the line passing through both charges and clearly explain your answers to the following: [You can answer this problem without any calculations]. Do not consider any points at infinite distance from the charges. [5 points](a) Find the location of all points, if any, where the electric potential is zero.(b) Find the location of all points, if any, where the electric field is zero.

Answers

Answer:

a. d/2 mid-way between the charges.

b. d/2 mid-way between the charges.

Explanation:

(a) Find the location of all points, if any, where the electric potential is zero.

Since the charges are of equal magnitude and opposite charge and separated by a distance, d, the electric potential due to the +Q charge is V = kQ/x and that due to the -Q charge is V' = -kQ/(d - x) where x is the point of zero electric potential.

The potential is zero when  V + V' = 0, and this can only be midway between the charges. This is shown below

So, kQ/x + [-kQ/(d - x)] = 0

kQ/x - kQ/(d - x) = 0

kQ/x = kQ/(d - x)

1/x = 1/(d - x)

(d - x) = x

d = x + x

d = 2x

x = d/2 which is mid-way between the charges.

(b) Find the location of all points, if any, where the electric field is zero.

Since the charges are of equal magnitude and opposite charge and separated by a distance, d, the electric field due to the +Q charge is E = kQ/x² and that due to the -Q charge is E' = -kQ/(d - x)² where x is the point of zero electric field.

The electric field is zero when  E + E' = 0 and this can only be midway between the charges. This is shown below.

So, kQ/x² + [-kQ/(d - x)²] = 0

kQ/x² - kQ/(d - x)² = 0

kQ/x² = kQ/(d - x)²

1/x² = 1/(d - x)²

(d - x)² = x²

d - x = ± x

d = x ± x

d = x - x or x + x

d = 0 or 2x

d = 0 or d = 2x

Since d ≠ 0, d = 2x ⇒ x = d/2 which is midway between the charges.

State three factors affecting pressure in liquids ​

Answers

Answer:

Density of liquid

Depth of liquid

Acceleration due to gravity

A 2.0-kilogram ball traveling north at 4.0 meters per second collides head on with a 1.0-kilogram ball traveling south at 8.0 meters per second. What is the magnitude of the total momentum of the two balls after collision?

Answers

Answer:

We know the momentum after the collision MUST be equal to the momentum BEFORE the collision.  

Momentum is a VECTOR quantity having both magnitude and direction.  The first ball has momentum P =m*v = 2*4 = 8 at 90degrees.  The second ball has momentum P = 1*8 = 8 at -90 or 270 degrees.  They sum to zero when you perform vector addition.

Explanation:

A ball is thrown vertically downward from the top of a 37.4-m-tall building. The ball passes the top of a window that is 15.4 m above the ground 2.00 s after being thrown. What is the speed of the ball as it passes the top of the window?

Answers

Answer:

v= 20.8 m/s

Explanation:

Assuming no other forces acting on the ball, from the instant that is thrown vertically downward, it's only accelerated by gravity, in this same direction, with a constant value of -9.8 m/s2  (assuming the ground level as the zero reference level and the upward direction as positive).In order to find the final speed 2.00 s after being thrown, we can apply the definition of acceleration, rearranging terms, as follows:

       [tex]v_{f} = v_{o} + a*t = v_{o} + g*t (1)[/tex]

We have the value of t, but since the ball was thrown, this means that it had an initial non-zero velocity v₀.Due to we know the value of the vertical displacement also, we can use the following kinematic equation in order to find the initial velocity v₀:

        [tex]\Delta y = v_{o} *t + \frac{1}{2} * a* t^{2} (2)[/tex]

where Δy = yf - y₀ = 15.4 m - 37.4 m = -22 m (3)Replacing by the values of Δy, a and t, we can solve for v₀ as follows:

       [tex]v_{o} = \frac{(\Delta y- \frac{1}{2} *a*t^{2})}{t} = \frac{-22m+19.6m}{2.00s} = -1.2 m/s (4)[/tex]

Replacing (4) , and the values of g and t in (1) we can find the value that we are looking for, vf:

       [tex]v_{f} = v_{o} + g*t = -1.2 m/s - (9.8m/s2*2.00s) = -20.8 m/s (5)[/tex]

Therefore, the speed of the ball (the magnitude of the velocity) as it passes the top of the window is 20.8 m/s.

A particle moves along the x-axis according to the equation (x=14-7t+t^2+t^3 ), where (x) in meter and (t) in seconds. At (t=7 sec) Find (a) The position of the particle (b) It’s velocity (c) It’s acceleration​

Answers

Answer:

jjnn ok jjjmkkmmkijnnkko

Cesar and Jill went to a field to play soccer. As the ball downward toward Jill, Jill used her foot to kick the ball and keep it in play. Cesar realized he could apply scientific principles to a soccer game. Which of the following best describes the scientific principle that Cesar observes as Jill kicks the ball?

A. An unbalanced force has no effect on the ball.
B. Gravity on the ball is equal to the force of friction.
C. A net force of zero changes the direction of the ball.
D. Unbalanced forces change the ball’s speed and the direction of motion.

Answers

Answer:

D

Explanation:



Describe effective communication strategies for gathering information, educating patients

Answers

Explanation:

Effective communication with patients will enable one to know the needs of the patient better as well as reducing the barriers to understanding each other for both parties.

To be an effective communicator while educating patients, the person must:

It is important to establish good rapport with the patient. By so doing they can trust you and let you in. Show empathy. Do not make them feel like you are judging them

Use proper body language. Make eye contacts and try to be on the same level as the patient so you can be face to face with them.make the interaction easier for them. You have to keep questions as well as your sentences short and moderate. Stay on topic and always make sure that concepts are clear to them.show respect. try not to speak with commands. Give the patient opportunity to make choices.be patient with them. Due to age or the nature of their illnesses, the patient may be slow in speech or movement. help them to move at their own pace by not rushing them.give them time to respond and ask questions. this will make communication more effective.you cause graphics where necessary or written instructions for the patient.

An iron nail becomes a permanent magnet if it is

Answers

if you stroke it an iron nail with a bar magnet the nail will become a permanent or long lasting magnet.

Hope it's perfect for you.

The picture to the right shows which wave behavior?

Answers

Answer:

It is refraction

Explanation:

I think is refraction

if a car travels 200 m to the east in 8.0 s what is the cars average velocity?

Answers

Answer:

25 m/s

Explanation:

200/8 = 25

toy car A drives with a steady force of 35N and covers 2000 m with fully charged battery. toy car B drives with a steady force of 80 N. how far would it be able to drive using the same fully charged battery as car A.​

Answers

The distance travelled by toy car B using the same fully charged battery as car A is 875 m

How to determine the energy of car AForce (F) = 35 NDistance of car A (d) = 2000 mEnergy (E) = ?

E = fd

E = 35 × 2000

E = 70000 J

How to determine the distance travelled by car BEnergy (E) = 70000 JForce (F) = 80 NDistance of car B =?

E = fd

70000 = 80 × Distance of car B

Divide both sides by 80

Distance of car B = 70000 / 80

Distance of car B = 875 m

Learn more about energy:

https://brainly.com/question/10703928

#SPJ1

Which graph best represents the greatest amount of work

Answers

Where are the graphs please they are not showing I really wanna help

coefficient of static friction formula

Answers

In coefficient of static friction, force is directly proportional to normal reaction, that means:

F=M N (consider M as coefficient of static friction)

now,

M=F/N

If you'll see the unit, then unit of F is Newton and unit of N is also Newton.

Therefore,

M= newton/newton

Therefore, Coefficient of static friction is unitless.

What did people assume Katherine was when she entered the room?

Answers

Answer: custodian,

Explanation: they never saw any colored women in the division before

Other Questions
the sum of two numbers is 51 and the difference is 13. what are the numbers Which of the following was supposed to increase support for communism and remove opposition to the communist party? Write the statement as a rate. Lilly records 65 movies onto 5 DVDs. Choose the correct answer below. 65 movies . 1 DVD 5 movies . 5 DVDs 65 DVDs OC 5 movies 65 DVDs OD 1 movie ****I RLY NEED HELP RN!!! ITS CHEMISTRY AND I AM SOOOO LOST. CAN SOMEONE PLZ PLZ PLZ HELP A GIRL OUT??? I WILL MARK BRAINLIEST, ETC!!!!!Balance the following chemical equations:1) AgNO3 + KCl AgCl + KNO32) H2O + SO3 H2SO43) KI + Cl2 KCl + I24) NaHCO3 Na2CO3 + H2O + CO25) Zn + HCl ZnCl2 + H26) BaCl2 + Na2SO4 BaSO4 + NaCl Click an item in the list or group of pictures at the bottom of the problem and, holding the button down, drag it into the correct position in the answer box. Release your mouse button when the item is place. If you change your mind, drag the item to the trashcan. Click the trashcan to clear all your answers.Assuming construction of the dotted lines, complete the following proof.Given: lines from P tangent to circle R at points A and BProve: PA = PB A _____________ is a person who has never married. Find the GCF of the terms of the polynomial. ANSWER WITH AN EXPLANATION please im bad at this 4p3 + 12p2 18pA. pB. 2pC. 4pD. 2p^3 A sports shop got a skateboard that cost them $80. In order to make aprofit, they decide to add a 37.5% markup. What is the retail price of theskateboard? He is tired because he has worked ___________ (hard)- adverb Read the list of elements given below.F, Cl, Br, IWhich of the following about the elements is true?A. All of them are highly reactive metalsB. All of them are highly reactive nonmetalsC. F and Cl are reactive nonmetals and Br and I are highly reactive metalsD. F and Cl are reactive metals and Br and I are highly reactive nonmetals The least multiple of 5 which is divisible by both 3 and 4 is - Find the perimeter of a square with sides that measure (2/3)g centimeters. How does folk songs reflect Nepalese culture and lifestyle? Which of the following wars generated famous American patriotic songs? O A. Revolutionary War O B. Civil War C. World War I O D. All of the above Cherry Avenue Park is in the shape of a rectangle. What is the area, in squaremiles, of Cherry Avenue Park? Common factors for 18 and 54 and 29 and 39 with a GCF for each pair I'll give brainlest What is an easy and accurate way that a graphic designer can increase the size of a digital shape while keeping the ratio of height to width the same? First make the height taller, and then drag the width by using ones eyes to try to make sure the proportions stay the same. Hold the key, such as Shift, that the program uses to make sure all dimensions are adjusted while dragging just one side. Open the shapes properties window, and type the height value multiplied by 2 and the width value multiplied by 3. First convert the shape into a photo file, and then digitally manipulate it in a program such as Adobe Photoshop. can someone help me witth these two questions Y= -4+9A.(3,-3)B.(2,3)C.(0,9)D.(2,1)E.(4,5) Divide x^2+7x+12 by x+3 using long division